LSAT and Law School Admissions Forum

Get expert LSAT preparation and law school admissions advice from PowerScore Test Preparation.

User avatar
 KwakuS
  • Posts: 35
  • Joined: Jun 03, 2021
|
#88697
Hello,

So for answer choice A, I notice that there is not a conclusion. It says that computer fraud cases take longer, and prosecutors are measured based on productivity, but it does not finish the argument and say "therefore, prosecutors are less likely to pursue computer fraud cases because they take longer..." or something like that. Is it good LSAT strategy for us to go ahead and draw that conclusion on this answer choice? Or should we not do that?
 Robert Carroll
PowerScore Staff
  • PowerScore Staff
  • Posts: 1783
  • Joined: Dec 06, 2013
|
#88760
KwakuS,

This is a Strengthen - Except question. The claim we're supporting is in the stimulus already. The answer choice should not have a conclusion - each incorrect answer choice should be evidence that helps support the claim already in the stimulus.

Note that adding the conclusion to "finish" the argument can't add any support to it anyway - a conclusion never adds support. The support comes entirely from the premises. How well those premises support the conclusion can depend on the conclusion made - if the conclusion goes well beyond the premises, the result could be an argument with a flaw, but that's not relevant here. The answer choice is not going to change the claim to be supported, which is the stimulus.

Robert Carroll
User avatar
 annabelle.swift
  • Posts: 54
  • Joined: Sep 01, 2021
|
#90526
Hi! I chose D, but it took me awhile because I was choosing between A and D.

I thought A would not support the stimulus because it specifically talks about computer-FRAUD cases, rather than computer crimes in general. Would A still support the stimulus since a lower rate of computer-fraud cases being brought before a jury would still result in a lower rate of computer crimes being brought before a jury?

Thanks!
User avatar
 evelineliu
PowerScore Staff
  • PowerScore Staff
  • Posts: 91
  • Joined: Sep 06, 2021
|
#90694
Hi Annabelle,

Reading the question stems tells us that four of the answer choices will strengthen the argument and one will not. The author's conclusion is that computer criminals are likely to avoid arrest and conviction even when their crimes are detected and reported. The author does not provide evidence to back up this claim.

(D) is out of scope. The argument deals with the prospects of arrest and conviction, and what happens after conviction (which is when sentence occurs) is not at issue here.

The other answer choices all relate to the unlikelihood of either arrest or conviction. (A) strengthens the argument by suggesting that because prosecutors respond to incentives to get the greatest number of convictions will not make prosecuting computer crimes a high priority. Computer-fraud cases are still computer cases!

Best,
Eveline
User avatar
 Hosseingold29
  • Posts: 5
  • Joined: Oct 29, 2022
|
#98308
I think E has nothing to do . bc of maybe the destruction of evidences will lead to imposing more burden on a perpetrator and isnt a favor for him ...

however D is too far than E .
THANKS :)
 Luke Haqq
PowerScore Staff
  • PowerScore Staff
  • Posts: 722
  • Joined: Apr 26, 2012
|
#98339
Hi Hosseingold29!

To address answer choices (D) and (E), let's start with the conclusion about crimes committed by computer: "the odds of being both arrested and convicted greatly favor the criminal." Given the question stem, four answer choices will strengthen the argument, and the correct answer will not strengthen it.

Answer choice (E) states, "The many police officers who are untrained in computers often inadvertently destroy the physical evidence of computer crime." This suggests that the odds of being arrested and convicted for computer crimes favor the criminal, since this answer choice is indicating that particular type of crime as a type where conditions make destruction of evidence more likely. Since those conditions favor the criminal, it strengthens the conclusion so can be eliminated as an answer choice.

Answer choice (D) states, "Computer criminals have rarely been sentenced to serve time in prison, because prisons are overcrowded with violent criminals and drug offenders." This doesn't get to the conclusion about the "odds of being arrested and convicted," because this answer choice is addressing something that comes after those two, namely, sentencing. Since this answer choice doesn't quite address the conclusion, it thus also doesn't strengthen it, making it the correct answer.

Get the most out of your LSAT Prep Plus subscription.

Analyze and track your performance with our Testing and Analytics Package.